頭の整理

頭の中を整えるために,色々と書き綴ります

合わせ鏡の問題

自作問題です.解答は下部に載せました.

 

問題

平面上に長さ  { \displaystyle L } の平行な2本の線分  { \displaystyle OA } と  { \displaystyle O' A' } がある.直線  { \displaystyle OA } 上にあり,点  { \displaystyle A } からの距離が  { \displaystyle h } である点を  { \displaystyle P } とする.点  { \displaystyle P } と線分  { \displaystyle O' A' } 上の点  { \displaystyle Q } を結ぶ線分を  { \displaystyle PQ } とする.線分  { \displaystyle PQ } の端点となる点  { \displaystyle Q } と点  { \displaystyle O' } との距離の最大値を  { \displaystyle M_0 } とし.このときの点  { \displaystyle Q } を  { \displaystyle Q_0 } とおく.直線  { \displaystyle O O' } と平行であり,かつ,点  { \displaystyle Q_0 } を通る直線を軸として,線分  { \displaystyle PQ_0 } と線対象な線分を 線分  { \displaystyle Q_0 Q_1 } とする.そして,線分  { \displaystyle O Q_1 } の長さを  { \displaystyle M_1 } とする.続いて,直線  { \displaystyle O O' } と平行であり,かつ,点  { \displaystyle Q_1 } を通る直線を軸として,線分  { \displaystyle Q_0 Q_1 } と線対象な線分を 線分  { \displaystyle Q_1 Q_2 } とする.そして,線分  { \displaystyle O' Q_2 } の長さを  { \displaystyle M_2 } とする.以後,同様の操作を  { \displaystyle M_n \geq 0 } を満たす範囲で繰り返す.このとき, { \displaystyle M_n } を求めよ.

 

 

 

 

 

 

 

 

 

 

 

 

 

 

解答

f:id:todayf0rmu1a:20191119185846p:plain

上図より,

 { \displaystyle M_n = L - nh \ \ \ \left( 0 \leq n \leq \frac{L}{h} \right) }

ここで, { \displaystyle n } は整数である.

 

 

「Amazon.co.jpアソシエイト」